LSAT and Law School Admissions Forum

Get expert LSAT preparation and law school admissions advice from PowerScore Test Preparation.

 Administrator
PowerScore Staff
  • PowerScore Staff
  • Posts: 8916
  • Joined: Feb 02, 2011
|
#23970
Complete Question Explanation

Strengthen-CE. The correct answer choice is (A)

Answer choice (A): This is the correct answer choice. This answer choice supports the claim that the North Sea can be unprecedentedly detrimental to other species’ populations as well. While this answer choice doesn’t completely justify the conclusion, it does seem to make it more likely that the unprecedented drop in the North Sea seal population was somehow related to the North Sea pollution.

Answer choice (B): This answer choice actually weakens the conclusion, since it would seem to make it less likely that the North Sea pollution could have such a profound effect on the immune systems of the seals.

Answer choice (C): This answer choice also slightly weakens the argument, since it would appear to downplay the level of pollution in the North Sea: If the North Sea pollution wasn’t hurting these fish, and wasn’t making them toxic to humans, then maybe the pollution isn’t so bad there.

Answer Choice (D): This answer is incorrect, because a breakdown of the species types in the area is irrelevant. We have information about the North Sea seal population as a whole. Knowing how many species inhabit the waters there neither increases nor reduces the strength of the argument in the stimulus.

Answer Choice (E): This answer weakens the argument that pollution was to blame for the reduced immunity, and offers the alternative explanation that it was a new disease, which the seals’ immune systems were unprepared to fight.
 GLMDYP
  • Posts: 100
  • Joined: Aug 19, 2013
|
#12457
Hi!
I don't understand why (A) is correct. Is it because that the pollution poisoned the shellfish and seabirds?
Thanks!
 David Boyle
PowerScore Staff
  • PowerScore Staff
  • Posts: 836
  • Joined: Jun 07, 2013
|
#12469
GLMDYP wrote:Hi!
I don't understand why (A) is correct. Is it because that the pollution poisoned the shellfish and seabirds?
Thanks!
Hello GLMDYP:

If this is the question beginning "A distemper virus has caused two-thirds of the seal population in the North Sea to die since May 1988…": then, yes, it is because the pollution poisoned the fish/birds, showing the North Sea is horribly polluted and could have poisoned the seals too (hurt their immune systems), to the point where they died en masse like the fish/birds.

Hope this helps,
David
 GLMDYP
  • Posts: 100
  • Joined: Aug 19, 2013
|
#12537
Thanks!
 Mkendz
  • Posts: 8
  • Joined: Feb 11, 2015
|
#18299
Question 14 stem asks to strengthen the explanation that "the severe pollution of the North Sea must have weakened the immune system of the seals so that they could no longer withstand the virus"
I ended up selecting A as my answer (which is the correct answer) only because I was able to determine that answers B-E were wrong.

I'm having trouble understanding why A is correct.

how does stating that Shellfish and seabirds have (unprecedentedly) seen steep declines in their population support the explanation ? couldn't there have been other causes other than pollution that have caused a decline in their populations ? (ex, what if volitile changes in water temp from global warming caused a decline in the shellfish population, and shellfish are the primary source of food for seabirds thereby (lack of food) resulting in a decline in the population of seabirds)

Some clarification on why A is correct would be great. Thanks !
 Nikki Siclunov
PowerScore Staff
  • PowerScore Staff
  • Posts: 1362
  • Joined: Aug 02, 2011
|
#18311
Hi Mkendz,

Thanks for the question. You're absolutely correct in that (A) does not rule out all alternative explanations for the observed phenomenon. However, it need not do so. You aren't being asked to Justify the conclusion here; your job is merely to strengthen it. So, if other marine animals are also experiencing unprecedented drops in their populations, it is even more likely that something that can potentially affect all animals - such as pollution - is responsible for the decline in the seal population. Indeed, if the latter explanation is to be believed, we would expect to see evidence that the seals aren't the only ones being affected (it would be weird for the pollution to only weaken their immune system, and no one else's).

Does that make sense? Let me know.

Thanks!
 whardy21
  • Posts: 48
  • Joined: Sep 30, 2018
|
#64242
I chose C. I thought that if fish were taken from the waters as a food source that contributed to another explanation for deaths. The conclusion says the explanation for deaths cannot rest here. I prephrased an answer choice that would strengthen the conclusion. I looked for an answer that could give another reason for the deaths.

Moot Point, I know its cause and effect (strengthen). I thought the conclusion led me to believe I needed an alternate cause to strengthen it. Weaken causal conclusions require an alternate cause for states effect. I just thought in particular, this conclusion needed me to find an alternate cause to strengthen the fact that the 1 explanation for the seals decline is not the only explanation. Please help!
 Brook Miscoski
PowerScore Staff
  • PowerScore Staff
  • Posts: 418
  • Joined: Sep 13, 2018
|
#64267
Whardy21,

You don't strengthen a cause-and-effect argument by finding a different cause--that weakens the relationship asserted by the stimulus. Instead, you would eliminate a competing cause, or you would find an answer that gives more reason to believe the cause-effect asserted by the stimulus. Another way you can strengthen a cause-effect claim is to show other ways in which the asserted cause is having the asserted effect.

(C) suggests that humans have eaten the seal's food so that starvation, not pollution, is the reason for the declining seal population.

(A) indicates that there is a widespread drop in shellfish and seabirds (not things seals eat, according to the stimulus). Thus, it avoids suggesting another cause (starvation), while the sudden drop in population suggests that the cause (pollution) is having a widespread effect on the population of wildlife in general.
 yournoona
  • Posts: 18
  • Joined: Mar 13, 2020
|
#74542
Hello
Although i did get this answer, i am having trouble identifying how this question is a strengthen type rather than being a must be true question.
Will all the if true, strongly supports the statement/explanation be strengthen type only?

Sometimes these also turn out to be the must be true. Please could you give me some indicators to identify between these two concepts. Also is the same given in the critical reasoning bible of gmat?
User avatar
 Dave Killoran
PowerScore Staff
  • PowerScore Staff
  • Posts: 5853
  • Joined: Mar 25, 2011
|
#74551
yournoona wrote:Hello
Although i did get this answer, i am having trouble identifying how this question is a strengthen type rather than being a must be true question.
Will all the if true, strongly supports the statement/explanation be strengthen type only?
It depends on the language. In this case, this fits the Second Family language model, hence it's Strengthen. Note the information flow: one of the answers will affect the stimulus. So, it's not Must!



yournoona wrote:Sometimes these also turn out to be the must be true. Please could you give me some indicators to identify between these two concepts. Also is the same given in the critical reasoning bible of gmat?
They actually don't randomly or "sometimes" turn out to be Must—there are specific reasons one would classify it as Must or Strengthen :-D This is all explained in any of our LR/CR books. Read the chapters on question stems (especially the Families), Must, and Strengthen. I can't easily summarize it here since it's pages worth of info, but the language tells you everything you need to know!

Get the most out of your LSAT Prep Plus subscription.

Analyze and track your performance with our Testing and Analytics Package.